0 oy
Sonsuz Toplamlar kategorisinde tarafından
$$\sum_{n=1}^\infty \frac{1}{2^{\ln n}}$$ toplamının yakınsaklığını inceleyiniz.

2 Cevaplar

0 oy
tarafından

Fikir:
Verilen toplamın yakınsaklığını ya da ıraksaklığını bildiğimiz bir toplam ile ilişkilendirerek bulmaya çalışacağız. 

Analiz:
Toplamı basitleştirmek için $e^{\ln n}=n$ eşitliğini kullanabiliriz. Bu şekilde toplamı terimleri $1/n$ olan ıraksak toplamdan büyük kılmış oluruz.

Direkt karşılaştırma testine aday terim:
$n$ pozitif bir tam sayı olmak üzere $2\le e$ olduğundan \begin{equation} \frac{1}{2^{\ln n}}\ge \frac{1}{e^{\ln n}}=\frac1n  \end{equation} eşitsizliği sağlanır.

Karşılaştırma yapacağımız toplamın ıraksaklığı:
$p=1\le 1$ olduğundan $$\displaystyle\sum_{n=1}^\infty \frac1{n}$$ toplamı  $p$-toplam testi gereği ıraksaktır.

Toplamın ıraksaklığı:
Bu pozitif terimli toplam ıraksak olduğundan, direkt karşılaştırma testi gereği, $$\displaystyle\sum_{n=1}^\infty \frac{1}{2^{\ln n}}$$ toplamı ıraksak olur.

0 oy
tarafından

Fikir:
Verilen toplamın yakınsaklığını ya da ıraksaklığını bildiğimiz bir toplam ile ilişkilendirerek bulmaya çalışacağız. 

Analiz:
Toplamı basitleştirmek için $2^{\ln n}=n^{\ln2}$ eşitliğini kullanabiliriz. Bu şekilde toplamı terimleri $1/n^{\ln2}$ olan ıraksak toplamı olarak yazmış oluruz.

Terim eşitliği:
$n$ pozitif bir tam sayı olmak üzere  \begin{align*}2^{\ln n}&=(e^{\ln 2})^{\ln n}\\[5pt]&=e^{\ln 2\cdot \ln n}\\[5pt]&=(e^{\ln n})^{\ln 2}\\[5pt]&=n^{\ln 2} \end{align*}eşitliği sağlanır.

Toplamın ıraksaklığı:
$p=\ln 2\le 1$ olduğundan $$\sum_{n=1}^\infty \frac{1}{2^{\ln n}}=\sum_{n=1}^\infty \frac1{n^{\ln2}}$$ toplamı  $p$-toplam testi gereği ıraksaktır.

...